Determine if $f(z) = log(e^z+1)$ is analytic and where












2












$begingroup$


I tried to substitute $z$ with $x+iy$ and then write it down as $f(z) = log(e^xcis(iy)+1) $ but it looks worse than the starting point. on original branch










share|cite|improve this question











$endgroup$








  • 1




    $begingroup$
    Analytic where? The composition of analytic functions is analytic. But you have to be careful with $ln$: what branch are you using?
    $endgroup$
    – Robert Israel
    Jan 2 at 20:16












  • $begingroup$
    @RobertIsrael Actually it's a warmup problem for my next week exam, and i'm hell of a newbie to this section, so i think the point is to determine where is analytical area of this function.
    $endgroup$
    – no0ob
    Jan 2 at 20:20












  • $begingroup$
    @RobertIsrael after reading my note i think your answer is $-pi lt theta le pi$
    $endgroup$
    – no0ob
    Jan 2 at 20:29








  • 1




    $begingroup$
    Look at previous comment. $u=ln(r), v=theta$.
    $endgroup$
    – herb steinberg
    Jan 3 at 2:20






  • 1




    $begingroup$
    $theta$ has two possible values. To get the correct one, use $sintheta = frac{e^xsiny}{r}$ and $costheta=frac{e^xcosy+1}{r}$..
    $endgroup$
    – herb steinberg
    Jan 3 at 2:31
















2












$begingroup$


I tried to substitute $z$ with $x+iy$ and then write it down as $f(z) = log(e^xcis(iy)+1) $ but it looks worse than the starting point. on original branch










share|cite|improve this question











$endgroup$








  • 1




    $begingroup$
    Analytic where? The composition of analytic functions is analytic. But you have to be careful with $ln$: what branch are you using?
    $endgroup$
    – Robert Israel
    Jan 2 at 20:16












  • $begingroup$
    @RobertIsrael Actually it's a warmup problem for my next week exam, and i'm hell of a newbie to this section, so i think the point is to determine where is analytical area of this function.
    $endgroup$
    – no0ob
    Jan 2 at 20:20












  • $begingroup$
    @RobertIsrael after reading my note i think your answer is $-pi lt theta le pi$
    $endgroup$
    – no0ob
    Jan 2 at 20:29








  • 1




    $begingroup$
    Look at previous comment. $u=ln(r), v=theta$.
    $endgroup$
    – herb steinberg
    Jan 3 at 2:20






  • 1




    $begingroup$
    $theta$ has two possible values. To get the correct one, use $sintheta = frac{e^xsiny}{r}$ and $costheta=frac{e^xcosy+1}{r}$..
    $endgroup$
    – herb steinberg
    Jan 3 at 2:31














2












2








2





$begingroup$


I tried to substitute $z$ with $x+iy$ and then write it down as $f(z) = log(e^xcis(iy)+1) $ but it looks worse than the starting point. on original branch










share|cite|improve this question











$endgroup$




I tried to substitute $z$ with $x+iy$ and then write it down as $f(z) = log(e^xcis(iy)+1) $ but it looks worse than the starting point. on original branch







complex-analysis analyticity






share|cite|improve this question















share|cite|improve this question













share|cite|improve this question




share|cite|improve this question








edited Jan 2 at 21:33







no0ob

















asked Jan 2 at 20:07









no0obno0ob

788




788








  • 1




    $begingroup$
    Analytic where? The composition of analytic functions is analytic. But you have to be careful with $ln$: what branch are you using?
    $endgroup$
    – Robert Israel
    Jan 2 at 20:16












  • $begingroup$
    @RobertIsrael Actually it's a warmup problem for my next week exam, and i'm hell of a newbie to this section, so i think the point is to determine where is analytical area of this function.
    $endgroup$
    – no0ob
    Jan 2 at 20:20












  • $begingroup$
    @RobertIsrael after reading my note i think your answer is $-pi lt theta le pi$
    $endgroup$
    – no0ob
    Jan 2 at 20:29








  • 1




    $begingroup$
    Look at previous comment. $u=ln(r), v=theta$.
    $endgroup$
    – herb steinberg
    Jan 3 at 2:20






  • 1




    $begingroup$
    $theta$ has two possible values. To get the correct one, use $sintheta = frac{e^xsiny}{r}$ and $costheta=frac{e^xcosy+1}{r}$..
    $endgroup$
    – herb steinberg
    Jan 3 at 2:31














  • 1




    $begingroup$
    Analytic where? The composition of analytic functions is analytic. But you have to be careful with $ln$: what branch are you using?
    $endgroup$
    – Robert Israel
    Jan 2 at 20:16












  • $begingroup$
    @RobertIsrael Actually it's a warmup problem for my next week exam, and i'm hell of a newbie to this section, so i think the point is to determine where is analytical area of this function.
    $endgroup$
    – no0ob
    Jan 2 at 20:20












  • $begingroup$
    @RobertIsrael after reading my note i think your answer is $-pi lt theta le pi$
    $endgroup$
    – no0ob
    Jan 2 at 20:29








  • 1




    $begingroup$
    Look at previous comment. $u=ln(r), v=theta$.
    $endgroup$
    – herb steinberg
    Jan 3 at 2:20






  • 1




    $begingroup$
    $theta$ has two possible values. To get the correct one, use $sintheta = frac{e^xsiny}{r}$ and $costheta=frac{e^xcosy+1}{r}$..
    $endgroup$
    – herb steinberg
    Jan 3 at 2:31








1




1




$begingroup$
Analytic where? The composition of analytic functions is analytic. But you have to be careful with $ln$: what branch are you using?
$endgroup$
– Robert Israel
Jan 2 at 20:16






$begingroup$
Analytic where? The composition of analytic functions is analytic. But you have to be careful with $ln$: what branch are you using?
$endgroup$
– Robert Israel
Jan 2 at 20:16














$begingroup$
@RobertIsrael Actually it's a warmup problem for my next week exam, and i'm hell of a newbie to this section, so i think the point is to determine where is analytical area of this function.
$endgroup$
– no0ob
Jan 2 at 20:20






$begingroup$
@RobertIsrael Actually it's a warmup problem for my next week exam, and i'm hell of a newbie to this section, so i think the point is to determine where is analytical area of this function.
$endgroup$
– no0ob
Jan 2 at 20:20














$begingroup$
@RobertIsrael after reading my note i think your answer is $-pi lt theta le pi$
$endgroup$
– no0ob
Jan 2 at 20:29






$begingroup$
@RobertIsrael after reading my note i think your answer is $-pi lt theta le pi$
$endgroup$
– no0ob
Jan 2 at 20:29






1




1




$begingroup$
Look at previous comment. $u=ln(r), v=theta$.
$endgroup$
– herb steinberg
Jan 3 at 2:20




$begingroup$
Look at previous comment. $u=ln(r), v=theta$.
$endgroup$
– herb steinberg
Jan 3 at 2:20




1




1




$begingroup$
$theta$ has two possible values. To get the correct one, use $sintheta = frac{e^xsiny}{r}$ and $costheta=frac{e^xcosy+1}{r}$..
$endgroup$
– herb steinberg
Jan 3 at 2:31




$begingroup$
$theta$ has two possible values. To get the correct one, use $sintheta = frac{e^xsiny}{r}$ and $costheta=frac{e^xcosy+1}{r}$..
$endgroup$
– herb steinberg
Jan 3 at 2:31










3 Answers
3






active

oldest

votes


















1












$begingroup$

You're using the branch of logarithm that is analytic on the complement of the left half-line $B = (-infty, 0]$. So your function is analytic at $z$ as long as $e^z+1 notin B$, i.e. $e^z notin (-infty, -1]$.



Now if $z = x + i y$ with $x, y$ real, $e^z = e^x e^{i y}$. For this to be in $B$, you need $y = (2n+1) pi$ with for integer $n$ (so that $e^{iy} = -1$) and $x ge 0$ (so $|e^z| = e^x ge 1$). Thus your function is analytic on the complement of
the half-lines $y = (2n+1) pi$, $x ge 0$.






share|cite|improve this answer









$endgroup$





















    1












    $begingroup$

    Summary of previous comments plus.



    Convert $e^z+1$ to a form $re^{itheta}$ Then $ln$ will be $ln(r)+itheta=u+iv$ for Cauchy-Riemann equations to check analytics. $e^z+1=e^xcosy+1+ie^xsiny$. Therefore $r^2=(e^xcosy+1)^2+(e^xsiny)^2$, $theta=arctan(frac{siny}{cosy+e^{−x}})$.



    For Cauchy-Riemann you could use $u=frac{ln(r^2)}{2}$. Also note that although $theta$ has two possible values, they differ by a constant ($pi$), so it doesn't effect the analysis.






    share|cite|improve this answer











    $endgroup$













    • $begingroup$
      I can't find relation between $r$ and $theta$ so i could write function as $u(r,theta) + iv(r,theta)$
      $endgroup$
      – no0ob
      Jan 3 at 12:41










    • $begingroup$
      $ln(e^z+1)=ln(r)+itheta$. Therefore $u=ln(r)=frac{ln(r^2)}{2}$ and $v=theta$
      $endgroup$
      – herb steinberg
      Jan 3 at 17:02










    • $begingroup$
      Aha Cauchy-Reimann is always correct for the $ln(r)+itheta$ now we have to check where does $ln(r)$ available for $r^2 = (e^xcos y+1)^2+(e^xsin y)^2$ and check domain of $arctan(frac{sin(y)}{cos(y)+e^{-x}}) $ and we are OK right?
      $endgroup$
      – no0ob
      Jan 3 at 19:58












    • $begingroup$
      I mean $ln(r)+itheta$ is analytic everywhere except $0$ and negative $x$ axis
      $endgroup$
      – no0ob
      Jan 3 at 20:05










    • $begingroup$
      This ends up as $z=x+ipi$ with $xle 0$.
      $endgroup$
      – herb steinberg
      Jan 3 at 21:12



















    0












    $begingroup$

    According to the determination of logarythm you saw, you should know on which domain the complex logarithm is holomorphic.
    The question would be : can the function exp(z) + 1 go outside of this domain ?






    share|cite|improve this answer









    $endgroup$









    • 1




      $begingroup$
      actually i better asked determine analytical area.
      $endgroup$
      – no0ob
      Jan 2 at 20:22










    • $begingroup$
      Then the question would be the following: for which z does exp(z) + 1 belong to the analytical area of the logarithm?
      $endgroup$
      – Cauchy is my master
      Jan 2 at 20:24










    • $begingroup$
      I think that's correct, and ln(z) is analytic in all of the plane but $xle0$ right? for the original branch
      $endgroup$
      – no0ob
      Jan 2 at 20:31






    • 1




      $begingroup$
      That depends on the determination of argument you chose, but I think you chose the usual one, so it's correct.
      $endgroup$
      – Cauchy is my master
      Jan 2 at 20:44










    • $begingroup$
      so now i should use function composition or sth? or is it possible to do it with the Cauchy-Reimann equation?
      $endgroup$
      – no0ob
      Jan 2 at 20:51












    Your Answer





    StackExchange.ifUsing("editor", function () {
    return StackExchange.using("mathjaxEditing", function () {
    StackExchange.MarkdownEditor.creationCallbacks.add(function (editor, postfix) {
    StackExchange.mathjaxEditing.prepareWmdForMathJax(editor, postfix, [["$", "$"], ["\\(","\\)"]]);
    });
    });
    }, "mathjax-editing");

    StackExchange.ready(function() {
    var channelOptions = {
    tags: "".split(" "),
    id: "69"
    };
    initTagRenderer("".split(" "), "".split(" "), channelOptions);

    StackExchange.using("externalEditor", function() {
    // Have to fire editor after snippets, if snippets enabled
    if (StackExchange.settings.snippets.snippetsEnabled) {
    StackExchange.using("snippets", function() {
    createEditor();
    });
    }
    else {
    createEditor();
    }
    });

    function createEditor() {
    StackExchange.prepareEditor({
    heartbeatType: 'answer',
    autoActivateHeartbeat: false,
    convertImagesToLinks: true,
    noModals: true,
    showLowRepImageUploadWarning: true,
    reputationToPostImages: 10,
    bindNavPrevention: true,
    postfix: "",
    imageUploader: {
    brandingHtml: "Powered by u003ca class="icon-imgur-white" href="https://imgur.com/"u003eu003c/au003e",
    contentPolicyHtml: "User contributions licensed under u003ca href="https://creativecommons.org/licenses/by-sa/3.0/"u003ecc by-sa 3.0 with attribution requiredu003c/au003e u003ca href="https://stackoverflow.com/legal/content-policy"u003e(content policy)u003c/au003e",
    allowUrls: true
    },
    noCode: true, onDemand: true,
    discardSelector: ".discard-answer"
    ,immediatelyShowMarkdownHelp:true
    });


    }
    });














    draft saved

    draft discarded


















    StackExchange.ready(
    function () {
    StackExchange.openid.initPostLogin('.new-post-login', 'https%3a%2f%2fmath.stackexchange.com%2fquestions%2f3059915%2fdetermine-if-fz-logez1-is-analytic-and-where%23new-answer', 'question_page');
    }
    );

    Post as a guest















    Required, but never shown

























    3 Answers
    3






    active

    oldest

    votes








    3 Answers
    3






    active

    oldest

    votes









    active

    oldest

    votes






    active

    oldest

    votes









    1












    $begingroup$

    You're using the branch of logarithm that is analytic on the complement of the left half-line $B = (-infty, 0]$. So your function is analytic at $z$ as long as $e^z+1 notin B$, i.e. $e^z notin (-infty, -1]$.



    Now if $z = x + i y$ with $x, y$ real, $e^z = e^x e^{i y}$. For this to be in $B$, you need $y = (2n+1) pi$ with for integer $n$ (so that $e^{iy} = -1$) and $x ge 0$ (so $|e^z| = e^x ge 1$). Thus your function is analytic on the complement of
    the half-lines $y = (2n+1) pi$, $x ge 0$.






    share|cite|improve this answer









    $endgroup$


















      1












      $begingroup$

      You're using the branch of logarithm that is analytic on the complement of the left half-line $B = (-infty, 0]$. So your function is analytic at $z$ as long as $e^z+1 notin B$, i.e. $e^z notin (-infty, -1]$.



      Now if $z = x + i y$ with $x, y$ real, $e^z = e^x e^{i y}$. For this to be in $B$, you need $y = (2n+1) pi$ with for integer $n$ (so that $e^{iy} = -1$) and $x ge 0$ (so $|e^z| = e^x ge 1$). Thus your function is analytic on the complement of
      the half-lines $y = (2n+1) pi$, $x ge 0$.






      share|cite|improve this answer









      $endgroup$
















        1












        1








        1





        $begingroup$

        You're using the branch of logarithm that is analytic on the complement of the left half-line $B = (-infty, 0]$. So your function is analytic at $z$ as long as $e^z+1 notin B$, i.e. $e^z notin (-infty, -1]$.



        Now if $z = x + i y$ with $x, y$ real, $e^z = e^x e^{i y}$. For this to be in $B$, you need $y = (2n+1) pi$ with for integer $n$ (so that $e^{iy} = -1$) and $x ge 0$ (so $|e^z| = e^x ge 1$). Thus your function is analytic on the complement of
        the half-lines $y = (2n+1) pi$, $x ge 0$.






        share|cite|improve this answer









        $endgroup$



        You're using the branch of logarithm that is analytic on the complement of the left half-line $B = (-infty, 0]$. So your function is analytic at $z$ as long as $e^z+1 notin B$, i.e. $e^z notin (-infty, -1]$.



        Now if $z = x + i y$ with $x, y$ real, $e^z = e^x e^{i y}$. For this to be in $B$, you need $y = (2n+1) pi$ with for integer $n$ (so that $e^{iy} = -1$) and $x ge 0$ (so $|e^z| = e^x ge 1$). Thus your function is analytic on the complement of
        the half-lines $y = (2n+1) pi$, $x ge 0$.







        share|cite|improve this answer












        share|cite|improve this answer



        share|cite|improve this answer










        answered Jan 3 at 4:42









        Robert IsraelRobert Israel

        330k23218473




        330k23218473























            1












            $begingroup$

            Summary of previous comments plus.



            Convert $e^z+1$ to a form $re^{itheta}$ Then $ln$ will be $ln(r)+itheta=u+iv$ for Cauchy-Riemann equations to check analytics. $e^z+1=e^xcosy+1+ie^xsiny$. Therefore $r^2=(e^xcosy+1)^2+(e^xsiny)^2$, $theta=arctan(frac{siny}{cosy+e^{−x}})$.



            For Cauchy-Riemann you could use $u=frac{ln(r^2)}{2}$. Also note that although $theta$ has two possible values, they differ by a constant ($pi$), so it doesn't effect the analysis.






            share|cite|improve this answer











            $endgroup$













            • $begingroup$
              I can't find relation between $r$ and $theta$ so i could write function as $u(r,theta) + iv(r,theta)$
              $endgroup$
              – no0ob
              Jan 3 at 12:41










            • $begingroup$
              $ln(e^z+1)=ln(r)+itheta$. Therefore $u=ln(r)=frac{ln(r^2)}{2}$ and $v=theta$
              $endgroup$
              – herb steinberg
              Jan 3 at 17:02










            • $begingroup$
              Aha Cauchy-Reimann is always correct for the $ln(r)+itheta$ now we have to check where does $ln(r)$ available for $r^2 = (e^xcos y+1)^2+(e^xsin y)^2$ and check domain of $arctan(frac{sin(y)}{cos(y)+e^{-x}}) $ and we are OK right?
              $endgroup$
              – no0ob
              Jan 3 at 19:58












            • $begingroup$
              I mean $ln(r)+itheta$ is analytic everywhere except $0$ and negative $x$ axis
              $endgroup$
              – no0ob
              Jan 3 at 20:05










            • $begingroup$
              This ends up as $z=x+ipi$ with $xle 0$.
              $endgroup$
              – herb steinberg
              Jan 3 at 21:12
















            1












            $begingroup$

            Summary of previous comments plus.



            Convert $e^z+1$ to a form $re^{itheta}$ Then $ln$ will be $ln(r)+itheta=u+iv$ for Cauchy-Riemann equations to check analytics. $e^z+1=e^xcosy+1+ie^xsiny$. Therefore $r^2=(e^xcosy+1)^2+(e^xsiny)^2$, $theta=arctan(frac{siny}{cosy+e^{−x}})$.



            For Cauchy-Riemann you could use $u=frac{ln(r^2)}{2}$. Also note that although $theta$ has two possible values, they differ by a constant ($pi$), so it doesn't effect the analysis.






            share|cite|improve this answer











            $endgroup$













            • $begingroup$
              I can't find relation between $r$ and $theta$ so i could write function as $u(r,theta) + iv(r,theta)$
              $endgroup$
              – no0ob
              Jan 3 at 12:41










            • $begingroup$
              $ln(e^z+1)=ln(r)+itheta$. Therefore $u=ln(r)=frac{ln(r^2)}{2}$ and $v=theta$
              $endgroup$
              – herb steinberg
              Jan 3 at 17:02










            • $begingroup$
              Aha Cauchy-Reimann is always correct for the $ln(r)+itheta$ now we have to check where does $ln(r)$ available for $r^2 = (e^xcos y+1)^2+(e^xsin y)^2$ and check domain of $arctan(frac{sin(y)}{cos(y)+e^{-x}}) $ and we are OK right?
              $endgroup$
              – no0ob
              Jan 3 at 19:58












            • $begingroup$
              I mean $ln(r)+itheta$ is analytic everywhere except $0$ and negative $x$ axis
              $endgroup$
              – no0ob
              Jan 3 at 20:05










            • $begingroup$
              This ends up as $z=x+ipi$ with $xle 0$.
              $endgroup$
              – herb steinberg
              Jan 3 at 21:12














            1












            1








            1





            $begingroup$

            Summary of previous comments plus.



            Convert $e^z+1$ to a form $re^{itheta}$ Then $ln$ will be $ln(r)+itheta=u+iv$ for Cauchy-Riemann equations to check analytics. $e^z+1=e^xcosy+1+ie^xsiny$. Therefore $r^2=(e^xcosy+1)^2+(e^xsiny)^2$, $theta=arctan(frac{siny}{cosy+e^{−x}})$.



            For Cauchy-Riemann you could use $u=frac{ln(r^2)}{2}$. Also note that although $theta$ has two possible values, they differ by a constant ($pi$), so it doesn't effect the analysis.






            share|cite|improve this answer











            $endgroup$



            Summary of previous comments plus.



            Convert $e^z+1$ to a form $re^{itheta}$ Then $ln$ will be $ln(r)+itheta=u+iv$ for Cauchy-Riemann equations to check analytics. $e^z+1=e^xcosy+1+ie^xsiny$. Therefore $r^2=(e^xcosy+1)^2+(e^xsiny)^2$, $theta=arctan(frac{siny}{cosy+e^{−x}})$.



            For Cauchy-Riemann you could use $u=frac{ln(r^2)}{2}$. Also note that although $theta$ has two possible values, they differ by a constant ($pi$), so it doesn't effect the analysis.







            share|cite|improve this answer














            share|cite|improve this answer



            share|cite|improve this answer








            edited Jan 3 at 17:06

























            answered Jan 3 at 4:35









            herb steinbergherb steinberg

            3,0782311




            3,0782311












            • $begingroup$
              I can't find relation between $r$ and $theta$ so i could write function as $u(r,theta) + iv(r,theta)$
              $endgroup$
              – no0ob
              Jan 3 at 12:41










            • $begingroup$
              $ln(e^z+1)=ln(r)+itheta$. Therefore $u=ln(r)=frac{ln(r^2)}{2}$ and $v=theta$
              $endgroup$
              – herb steinberg
              Jan 3 at 17:02










            • $begingroup$
              Aha Cauchy-Reimann is always correct for the $ln(r)+itheta$ now we have to check where does $ln(r)$ available for $r^2 = (e^xcos y+1)^2+(e^xsin y)^2$ and check domain of $arctan(frac{sin(y)}{cos(y)+e^{-x}}) $ and we are OK right?
              $endgroup$
              – no0ob
              Jan 3 at 19:58












            • $begingroup$
              I mean $ln(r)+itheta$ is analytic everywhere except $0$ and negative $x$ axis
              $endgroup$
              – no0ob
              Jan 3 at 20:05










            • $begingroup$
              This ends up as $z=x+ipi$ with $xle 0$.
              $endgroup$
              – herb steinberg
              Jan 3 at 21:12


















            • $begingroup$
              I can't find relation between $r$ and $theta$ so i could write function as $u(r,theta) + iv(r,theta)$
              $endgroup$
              – no0ob
              Jan 3 at 12:41










            • $begingroup$
              $ln(e^z+1)=ln(r)+itheta$. Therefore $u=ln(r)=frac{ln(r^2)}{2}$ and $v=theta$
              $endgroup$
              – herb steinberg
              Jan 3 at 17:02










            • $begingroup$
              Aha Cauchy-Reimann is always correct for the $ln(r)+itheta$ now we have to check where does $ln(r)$ available for $r^2 = (e^xcos y+1)^2+(e^xsin y)^2$ and check domain of $arctan(frac{sin(y)}{cos(y)+e^{-x}}) $ and we are OK right?
              $endgroup$
              – no0ob
              Jan 3 at 19:58












            • $begingroup$
              I mean $ln(r)+itheta$ is analytic everywhere except $0$ and negative $x$ axis
              $endgroup$
              – no0ob
              Jan 3 at 20:05










            • $begingroup$
              This ends up as $z=x+ipi$ with $xle 0$.
              $endgroup$
              – herb steinberg
              Jan 3 at 21:12
















            $begingroup$
            I can't find relation between $r$ and $theta$ so i could write function as $u(r,theta) + iv(r,theta)$
            $endgroup$
            – no0ob
            Jan 3 at 12:41




            $begingroup$
            I can't find relation between $r$ and $theta$ so i could write function as $u(r,theta) + iv(r,theta)$
            $endgroup$
            – no0ob
            Jan 3 at 12:41












            $begingroup$
            $ln(e^z+1)=ln(r)+itheta$. Therefore $u=ln(r)=frac{ln(r^2)}{2}$ and $v=theta$
            $endgroup$
            – herb steinberg
            Jan 3 at 17:02




            $begingroup$
            $ln(e^z+1)=ln(r)+itheta$. Therefore $u=ln(r)=frac{ln(r^2)}{2}$ and $v=theta$
            $endgroup$
            – herb steinberg
            Jan 3 at 17:02












            $begingroup$
            Aha Cauchy-Reimann is always correct for the $ln(r)+itheta$ now we have to check where does $ln(r)$ available for $r^2 = (e^xcos y+1)^2+(e^xsin y)^2$ and check domain of $arctan(frac{sin(y)}{cos(y)+e^{-x}}) $ and we are OK right?
            $endgroup$
            – no0ob
            Jan 3 at 19:58






            $begingroup$
            Aha Cauchy-Reimann is always correct for the $ln(r)+itheta$ now we have to check where does $ln(r)$ available for $r^2 = (e^xcos y+1)^2+(e^xsin y)^2$ and check domain of $arctan(frac{sin(y)}{cos(y)+e^{-x}}) $ and we are OK right?
            $endgroup$
            – no0ob
            Jan 3 at 19:58














            $begingroup$
            I mean $ln(r)+itheta$ is analytic everywhere except $0$ and negative $x$ axis
            $endgroup$
            – no0ob
            Jan 3 at 20:05




            $begingroup$
            I mean $ln(r)+itheta$ is analytic everywhere except $0$ and negative $x$ axis
            $endgroup$
            – no0ob
            Jan 3 at 20:05












            $begingroup$
            This ends up as $z=x+ipi$ with $xle 0$.
            $endgroup$
            – herb steinberg
            Jan 3 at 21:12




            $begingroup$
            This ends up as $z=x+ipi$ with $xle 0$.
            $endgroup$
            – herb steinberg
            Jan 3 at 21:12











            0












            $begingroup$

            According to the determination of logarythm you saw, you should know on which domain the complex logarithm is holomorphic.
            The question would be : can the function exp(z) + 1 go outside of this domain ?






            share|cite|improve this answer









            $endgroup$









            • 1




              $begingroup$
              actually i better asked determine analytical area.
              $endgroup$
              – no0ob
              Jan 2 at 20:22










            • $begingroup$
              Then the question would be the following: for which z does exp(z) + 1 belong to the analytical area of the logarithm?
              $endgroup$
              – Cauchy is my master
              Jan 2 at 20:24










            • $begingroup$
              I think that's correct, and ln(z) is analytic in all of the plane but $xle0$ right? for the original branch
              $endgroup$
              – no0ob
              Jan 2 at 20:31






            • 1




              $begingroup$
              That depends on the determination of argument you chose, but I think you chose the usual one, so it's correct.
              $endgroup$
              – Cauchy is my master
              Jan 2 at 20:44










            • $begingroup$
              so now i should use function composition or sth? or is it possible to do it with the Cauchy-Reimann equation?
              $endgroup$
              – no0ob
              Jan 2 at 20:51
















            0












            $begingroup$

            According to the determination of logarythm you saw, you should know on which domain the complex logarithm is holomorphic.
            The question would be : can the function exp(z) + 1 go outside of this domain ?






            share|cite|improve this answer









            $endgroup$









            • 1




              $begingroup$
              actually i better asked determine analytical area.
              $endgroup$
              – no0ob
              Jan 2 at 20:22










            • $begingroup$
              Then the question would be the following: for which z does exp(z) + 1 belong to the analytical area of the logarithm?
              $endgroup$
              – Cauchy is my master
              Jan 2 at 20:24










            • $begingroup$
              I think that's correct, and ln(z) is analytic in all of the plane but $xle0$ right? for the original branch
              $endgroup$
              – no0ob
              Jan 2 at 20:31






            • 1




              $begingroup$
              That depends on the determination of argument you chose, but I think you chose the usual one, so it's correct.
              $endgroup$
              – Cauchy is my master
              Jan 2 at 20:44










            • $begingroup$
              so now i should use function composition or sth? or is it possible to do it with the Cauchy-Reimann equation?
              $endgroup$
              – no0ob
              Jan 2 at 20:51














            0












            0








            0





            $begingroup$

            According to the determination of logarythm you saw, you should know on which domain the complex logarithm is holomorphic.
            The question would be : can the function exp(z) + 1 go outside of this domain ?






            share|cite|improve this answer









            $endgroup$



            According to the determination of logarythm you saw, you should know on which domain the complex logarithm is holomorphic.
            The question would be : can the function exp(z) + 1 go outside of this domain ?







            share|cite|improve this answer












            share|cite|improve this answer



            share|cite|improve this answer










            answered Jan 2 at 20:19









            Cauchy is my masterCauchy is my master

            413




            413








            • 1




              $begingroup$
              actually i better asked determine analytical area.
              $endgroup$
              – no0ob
              Jan 2 at 20:22










            • $begingroup$
              Then the question would be the following: for which z does exp(z) + 1 belong to the analytical area of the logarithm?
              $endgroup$
              – Cauchy is my master
              Jan 2 at 20:24










            • $begingroup$
              I think that's correct, and ln(z) is analytic in all of the plane but $xle0$ right? for the original branch
              $endgroup$
              – no0ob
              Jan 2 at 20:31






            • 1




              $begingroup$
              That depends on the determination of argument you chose, but I think you chose the usual one, so it's correct.
              $endgroup$
              – Cauchy is my master
              Jan 2 at 20:44










            • $begingroup$
              so now i should use function composition or sth? or is it possible to do it with the Cauchy-Reimann equation?
              $endgroup$
              – no0ob
              Jan 2 at 20:51














            • 1




              $begingroup$
              actually i better asked determine analytical area.
              $endgroup$
              – no0ob
              Jan 2 at 20:22










            • $begingroup$
              Then the question would be the following: for which z does exp(z) + 1 belong to the analytical area of the logarithm?
              $endgroup$
              – Cauchy is my master
              Jan 2 at 20:24










            • $begingroup$
              I think that's correct, and ln(z) is analytic in all of the plane but $xle0$ right? for the original branch
              $endgroup$
              – no0ob
              Jan 2 at 20:31






            • 1




              $begingroup$
              That depends on the determination of argument you chose, but I think you chose the usual one, so it's correct.
              $endgroup$
              – Cauchy is my master
              Jan 2 at 20:44










            • $begingroup$
              so now i should use function composition or sth? or is it possible to do it with the Cauchy-Reimann equation?
              $endgroup$
              – no0ob
              Jan 2 at 20:51








            1




            1




            $begingroup$
            actually i better asked determine analytical area.
            $endgroup$
            – no0ob
            Jan 2 at 20:22




            $begingroup$
            actually i better asked determine analytical area.
            $endgroup$
            – no0ob
            Jan 2 at 20:22












            $begingroup$
            Then the question would be the following: for which z does exp(z) + 1 belong to the analytical area of the logarithm?
            $endgroup$
            – Cauchy is my master
            Jan 2 at 20:24




            $begingroup$
            Then the question would be the following: for which z does exp(z) + 1 belong to the analytical area of the logarithm?
            $endgroup$
            – Cauchy is my master
            Jan 2 at 20:24












            $begingroup$
            I think that's correct, and ln(z) is analytic in all of the plane but $xle0$ right? for the original branch
            $endgroup$
            – no0ob
            Jan 2 at 20:31




            $begingroup$
            I think that's correct, and ln(z) is analytic in all of the plane but $xle0$ right? for the original branch
            $endgroup$
            – no0ob
            Jan 2 at 20:31




            1




            1




            $begingroup$
            That depends on the determination of argument you chose, but I think you chose the usual one, so it's correct.
            $endgroup$
            – Cauchy is my master
            Jan 2 at 20:44




            $begingroup$
            That depends on the determination of argument you chose, but I think you chose the usual one, so it's correct.
            $endgroup$
            – Cauchy is my master
            Jan 2 at 20:44












            $begingroup$
            so now i should use function composition or sth? or is it possible to do it with the Cauchy-Reimann equation?
            $endgroup$
            – no0ob
            Jan 2 at 20:51




            $begingroup$
            so now i should use function composition or sth? or is it possible to do it with the Cauchy-Reimann equation?
            $endgroup$
            – no0ob
            Jan 2 at 20:51


















            draft saved

            draft discarded




















































            Thanks for contributing an answer to Mathematics Stack Exchange!


            • Please be sure to answer the question. Provide details and share your research!

            But avoid



            • Asking for help, clarification, or responding to other answers.

            • Making statements based on opinion; back them up with references or personal experience.


            Use MathJax to format equations. MathJax reference.


            To learn more, see our tips on writing great answers.




            draft saved


            draft discarded














            StackExchange.ready(
            function () {
            StackExchange.openid.initPostLogin('.new-post-login', 'https%3a%2f%2fmath.stackexchange.com%2fquestions%2f3059915%2fdetermine-if-fz-logez1-is-analytic-and-where%23new-answer', 'question_page');
            }
            );

            Post as a guest















            Required, but never shown





















































            Required, but never shown














            Required, but never shown












            Required, but never shown







            Required, but never shown

































            Required, but never shown














            Required, but never shown












            Required, but never shown







            Required, but never shown







            Popular posts from this blog

            How do I know what Microsoft account the skydrive app is syncing to?

            When does type information flow backwards in C++?

            Grease: Live!